Középiskolai Matematikai és Fizikai Lapok
Informatika rovattal
Kiadja a MATFUND Alapítvány
Már regisztráltál?
Új vendég vagy?

A B. 4550. feladat (2013. május)

B. 4550. Igazoljuk, hogy minden 4-nél nagyobb 2-hatvány előáll a2+7b2 alakban, ahol a és b pozitív páratlan számok.

Javasolta: Williams Kada Kálmán (Szeged)

(6 pont)

A beküldési határidő 2013. június 10-én LEJÁRT.


Megoldási ötlet: Keressünk rekurzív előállítást.

Megoldás: Könnyen ellenőrizhető, hogy

 
2(a^2+7b^2) 
= \left|\frac{a\pm 7b}2\right|^2 + 7\left|\frac{a\mp b}2\right|^2.
(1)

(Ez az azonosság az (a+b\sqrt7i)(c+d\sqrt7i)=(ac-7bd)+(ad+bc)\sqrt7i azonosságból származik.)

Az szükséges (a,b) párokat rekurzívan adjuk meg. Minden k\ge3-hoz konstruálunk egy páratlan pozitív egészekből álló (ak,bk) párt, amire ak2+7bk2=2k.

Legyen a3=b3=1. Ezekre valóban teljesül, hogy a32+7b33=23.

Ha az (ak,bk) számpárt már definiáltuk, akkor legyen


\matrix{
a_{k+1}=\dfrac{|a_k-7b_k|}2, & b_{k+1} = \dfrac{a_k+b_k}2 &
{\rm ha~} 
a_k\equiv b_k \pmod4; \cr
a_{k+1}=\dfrac{a_k+7b_k}2, & b_{k+1} = \dfrac{|a_k-b_k|}2 &
{\rm ha~} a_k\equiv -b_k \pmod4. \cr
}

Mivel ak és bk páratlan, a definíció értelmes, a kapott számok valóban egészek, és nem lehetnek negatívak. Még azt kell ellenőriznünmk, hogy páratlanok.

Ha a_k\equiv b_k\equiv \pm1 \pmod4, akkor a_k-7b_k\equiv a_k+b_k\equiv 2\pmod4, ezért a_{k+1}=\dfrac{|a_k-7b_k|}2 és b_{k+1} = \dfrac{a_k+b_k}2 is páratlan. Hasonlóan, ha a_k\equiv -b_k\equiv \pm1 \pmod4, akkor a_k+7b_k\equiv a_k-b_k\equiv 2\pmod4, ezért a_{k+1}=\dfrac{a_k+7b_k}2 és b_{k+1} = \dfrac{|a_k-b_k|}2 is páratlan.

Ezzel minden k\ge3-ra konstruáltunk egy megfelelő számpárt.

Megjegyzések. 1. A fenti sorozatokat közvetlenül is megadhatjuk.

Defniáljuk az An, Bn számokat (n=0,1,2,...) a következőképpen:


\left(\frac{1\pm\sqrt7i}2\right)^n = \frac{A_n\pm B_n\sqrt7i}2.

Ekkor A0=2, A1=1, A2=-3, ... és B0=0, B1=1, B2=1, ..., teljesülnek az An+1=An-2An-1 és Bn+1=Bn-2Bn-1 összefüggések (ezekből leolvasható, hogy n\ge1 esetén An és Bn is páratlan), továbbá


A_n^2+7B_n^2 
= 4\cdot \frac{A_n+B_n\sqrt7i}2 \cdot \frac{A_n-B_n\sqrt7i}2
= 4\cdot 
\left(\frac{1+\sqrt7i}2\right)^n \left(\frac{1-\sqrt7i}2\right)^n = 2^{n+2}.

Ezért az ak=|Ak-2|, bk=|Bk-2| választás megfelelő.

2. A keresett számsorozat egyértelmű. Ez az előző megjegyzés alapján is igazolható, de abból is bizonyítható, hogy az x+y\frac{1\sqrt7i}2 alakú számok körében, ahol x,y egészek, igaz a számelmélet alaptétele: minden x+y\frac{1\sqrt7i}2 alakú szám lényegében egyértelműen írható fel irreducibilis elemek szorzataként.


Statisztika:

31 dolgozat érkezett.
6 pontot kapott:Balogh Tamás, Bereczki Zoltán, Csépai András, Csernák Tamás, Dinev Georgi, Fehér Zsombor, Gyulai-Nagy Szuzina, Janzer Barnabás, Janzer Olivér, Kovács 972 Márton, Kúsz Ágnes, Lelkes János, Maga Balázs, Nagy Róbert, Sagmeister Ádám, Seress Dániel, Szabó 789 Barnabás, Szabó 928 Attila, Tossenberger Tamás, Venczel Tünde, Williams Kada, Zilahi Tamás.
5 pontot kapott:Di Giovanni Márk, Fonyó Viktória, Herczeg József, Kabos Eszter, Petrényi Márk, Török Tímea.
0 pontot kapott:1 versenyző.
Nem versenyszerű:2 dolgozat.

A KöMaL 2013. májusi matematika feladatai